2014 AMC 10A Problems/Problem 11

Revision as of 22:51, 6 February 2014 by TheMaskedMagician (talk | contribs) (Created page with "==Problem== A customer who intends to purchase an appliance has three coupons, only one of which may be used: Coupon 1: <math>10\%</math> off the listed price if the listed pri...")
(diff) ← Older revision | Latest revision (diff) | Newer revision → (diff)

Problem

A customer who intends to purchase an appliance has three coupons, only one of which may be used:

Coupon 1: $10\%$ off the listed price if the listed price is at least $\textdollar50$

Coupon 2: $\textdollar 20$ off the listed price if the listed price is at least $\textdollar100$

Coupon 3: $18\%$ off the amount by which the listed price exceeds $\textdollar100$

For which of the following listed prices will coupon $1$ offer a greater price reduction than either coupon $2$ or coupon $3$?

$\textbf{(A) }\textdollar179.95\qquad \textbf{(B) }\textdollar199.95\qquad \textbf{(C) }\textdollar219.95\qquad \textbf{(D) }\textdollar239.95\qquad \textbf{(E) }\textdollar259.95\qquad$